« first day (4509 days earlier)      last day (522 days later) » 

4:21 AM
@TedShifrin @TedShifrin sir this is just a question from spivak
 
4:38 AM
But you didn’t state the actual problem, did you? What was the actual question — not “solve”?
 
5:18 AM
phbtbht
 
5:30 AM
Munchkin has you wheezing?
 
5:40 AM
she's annoyingly less sick than i am.
 
How dare she?
 
6:00 AM
given something like $\int_0^2 \int_{\frac{y}{2}}^{1} {e^{x}}^{2} dxdy $ in order to sketch this, I've drew in the functions & values for the bounds. Without evaluating what else do I do again?
 
yeah, and she just got out of bed at almost 10 pm to rant to me about something.
obliv, i am not sure what the instructions were, but as far as 'sketch this' i think sketching the region of integration is appropriate. if you actually evaluate that thing, it would be a number, which isn't very fun to sketch.
[draws number line with point on it labeled 'the value of the integral']
 
why isn't the chatjax working?
there we go
dang so I have to know what e^x^2 looks like
I'm so bad at memorizing these things.
 
uh, i would be mildly surprised if they wanted you to do that. it's more common to ask just for a sketch of the region of integration (which would be the same no matter what the integrand is)
 
Oh okay that's what I did, I was afraid the integrand actually influenced the area.
 
if they really want a picture of the graph of z = e^(x^2) over the region of integration, presumably accuracy is not that big of a deal and basically any increasing concave function of x would give the general spirit.
 
6:05 AM
The just wanted the region R sketched so yeah just the bounds.
12 points secured. This test actually destroyed me though, I have no business passing it.
I spent 1 hour on 1 problem because of the dang cylindrical coordinates. I can't for the life of me do cylindrical coordinates.
I can do conversions from rect. to cyl/spherical pretty handily but when just given a solid by bounded graphs it's tuff.
 
well, if just passing is enough to get you where you want to go, i hope you at least pass.
i only taught multivar a few times, but even solid A students would goof up quite a bit in exam settings. there are too many things that can potentially go wrong.
it's really hard to write problems that don't invite mishaps that complicate everything, without giving just incredibly easy questions that chatGPT could do.
 
Let Q be the solid bounded by $x^2+z^2=1, y=x, x=0$ in the first octant. Set up the integral of its volume and sketch it. So this problem gave me a headache
I ended up using up too much time so I just set up two integrals for it which is probably correct but not the way he intended.
 
another day, another cylinder.
 
@Obliv no. Only the region needs to be drawn.
 
wait what does x=0 mean in R^3
for a bound..
oh it's the yz plane?
 
6:18 AM
I was talking about the double integral.
Yes, to your last question.
 
I did two cylinders $\frac{1}{2}\int_0^1\int_{0}^{\frac{\pi}{2}}\int_{0}^{1}rdrd\theta dz + \int_{0}^{\infty}\int_{0}^{\frac{\pi}{2}}\int_{0}^{1}rdrd\theta dz$ but looking back on it it should have been 1/8 and 1/4 in front of the integrals
 
Total nonsense.
 
yep, but I had to write something lol
Do these types of problems have the same setup/approach
 
Cylindrical is ok, but cartesian is fine.
 
 
6:22 AM
Put $z$ on the inside. You see the projection on the xy-plane, so $dydx$ should be outside.
That picture doesn’t help me. I draw $yz$ in the usual $xy$ positions, with $x$ coning out of the paper/board. I did several problems like this in my lectures.
 
Do you put Y on the vertical axis or horizontal?
So I just have the graph of a circle with x-axis in the center.
oops nvm I thought it was y^2+z^2=1
I will definitely take a look.
 
$z$ vertical
 
6:41 AM
were your lectures at georgia for undergrad students?
rifling through it a bit I noticed that my multivar course doesn't cover like half of the stuff on calculus alone
 
1st and 2nd year, but a very advanced course. However, the standard computational stuff (plus a lot of linear algebra) is in there. I’m suggesting you watch the computational examples, not the theory.
 
is there a way to write this notation in latex
I feel as though it would be neat
 
i dunno if it's in chatjax but the pmatrix environment is pretty good for more general versions of that
 
Is it true that for a continuous function that x < y implies f(x) < f(y)? We can also assume that the function is bounded. I was reading the definition of a lower and uppoer riemann integral which was given using supremum and infimum of a continuous bounded function from a closed subset of R to R. I haven't yet studied continuous functions much. So, was just wondering if that is the case.
 
let's see $f \begin{pmatrix} x \\ y \end{pmatrix}$
pmatrix might not be the right environment to get the proper spacing for its use as an argument to a function
 
6:51 AM
that was a lot of writing to pull that off though..
$\int_S f\begin{pmatrix} x \\ y \end{pmatrix}dA$
 
Never mind, clearly f(x) = x^2 is a counterexample for -2 < 2. But is it true that $x \le y$ implies $f(x)\le f(y)$?
 
yeah it's a bit big.
 
seeker: it might help to think about what the desired condition means graphically, and what continuity and boundedness mean graphically
 
how would that be a counter example? -2 is < 2 and -2 < 4
 
he had strict inequality in the first example
 
6:54 AM
oh
 
@leslietownes Graphically this seems to be true and I can't really find a counterexample. But I will try to think about it!
 
constant functions would also be counterexamples to that, but not to the one with non-strict inequality
seeker: consider the graph of f(x) = -x
or -arctan(x) if you want a bounded example on all of R
or sin(x) or you name it, any one example will have features that maybe distract from the general phenomenon
 
@leslietownes Right, that makes sense. Would adding the hypothesis that the function be non-negative make the statement true.
 
no, for example, -arctan(x) + 100
again, think graphically, vertically shifting a graph up or down is not going to change continuity or whether the property you want to hold actually does hold, but it can be used to change whether a bounded function is or isn't nonnegative
 
@leslietownes Right. Thanks for the help!
 
6:58 AM
found the example in Ted's book. truly blessed.
 
7:36 AM
wow, you got an edition that incorporates viete's algebraic notation
 
7:55 AM
There is this lemma that if 0≤a<b is true for all b>0, then a must be 0. I know one very short proof, but here is another attempt i saw online: b-a>0 which means a-b<0. Find two numbers n€N and c€W such that nc=a, we get nc-b<0 which is not true from archimedian principle. Is this proof correct to conclude nc must be zero?
 
8:24 AM
how does a-b < 0 imply that nc-b < 0 is not true?
@nickbros123 if a-b < 0, and nc = a, then a-b < 0 is the same as nc - b < 0
 
The proof is quoting the archimedean property, saying there always exists some n so that na-b>,0
 
if nc = a, there is nothing different between a-b<0 and nc-b<0
and the statement that there always exists some n so that na - b > 0 does not seem to me to be related to what you said, that nc - b < 0
because you chose a new c and a new n
 
9:03 AM
@nickbros123

Let x be a non-negative real number such that, given any real number r, we have that x is either at 0 or between 0 and x.

Suppose x is greater than 0. Clearly, there is a real number r_1 between 0 and x. But this contradicts our definition that, if x is not at 0, then for any real number, x is between 0 and that real number. Therefore, x cannot be greater than 0, and since it was either greater than 0 or at 0, x is at 0. Since both x is r_1 were arbitrary non-negative real numbers, this means that this holds if x is any real number defined as above.
@nickbros123 it helps to clearly distinguish between a statement that contains a quantified variable, and the instantiated variable within the proof
 
 
3 hours later…
11:43 AM
@shintuku this free use of different variables doesn't seem to bother my professor, as long as i establish how the variable change came into picture
@shintuku @shintuku you are right, but my initial question was, was the proof correct, atleast logically if not in a mathematically written way?
 
the proof you mentioned hinges on saying that a - b < 0 implies that nc - b < 0 is a contradiction, but that makes no sense if nc = a, making those two statements are exactly the same
 
Ohh ok I understand you now
Ah there is an inherent flaw in that argument, just realised
Thank you @shintuku
 
np!
 
My teacher just exposed me to archimedean principle, and this lemma. Where do I find problems pertaining to usage of these, Spivak doesn't talk about this, as far as I have come.
I feel like I understand more of the theorem if i do problems using it
 
this book is published for free by the authors: classicalrealanalysis.info/documents/…
it has exercises on the archimedean principle on page 17
 
12:05 PM
@shintuku it seems that one of the authors of this book happens to be an mse user as well.
 
cool!!
 
12:32 PM
@shintuku thanks !
 
12:59 PM
heyo! (:
 
1:10 PM
Does it hold that a function is monotonic increasing on an interval if $f'(x) \geq 0$ on that interval?
Or is it only $f'(x) > 0$ for strict monotonic increasing, but for just monotonic increasing, there isn't such theorem?
 
1:58 PM
@ILikeMathematics what is your question?
if $f' \ge 0$ on an interval then $f$ is non decreasing on that interval.
if $f'>0$ then $f$ is strictly increasing.
but $x \mapsto x^3$ is strictly increasing but the slope is zero at $x=0$.
 
2:35 PM
information geometry is essentially geometry that tells you information about a state of affairs
at each point on our statistical manifold we have a given hypothesis about something
 
2:49 PM
>statistical manifold
dear god
 
3:11 PM
Is it possible to prove Schwarz inequality using amgm inequality?
I mean more than proving, is it possible to get to the Schwartz inequality
 
 
2 hours later…
5:14 PM
'more than proving'? does math.stackexchange.com/questions/4092148/… help?
my gut feeling is that all of those fundamental inequalities can probably be proved in terms of one another (assuming that you have the one you're assuming in an appropriately general form, e.g. for arbitrary tuples of numbers)
 
I believe you can get the simple AMGM inequality from CS (I think that’s an exercise in my diff geo text), but I doubt vice versa.
Only for 2. How would we get $n$th root from CS? Hölder, yes.
 
ted i dunno those contest kids can come up with all kinds of tricks
 
I don’t see Hölder working, actually. … Well, that’s true, too.
How’s the invalid?
 
feeling a little better this morning.
 
5:35 PM
Well, I guess having Munchkin is worse than being a party animal.
 
yes, i think this is a fair assessment
 
5:56 PM
@leslietownes @leslietownes thank you for the link. It was helpful
I was thinking of doing the derivation myself, using the a<√ab<a+b/2<b property. Seemed to go endless
 
Go endless?
 
Well I drew conclusions on and on for a few pages and reached a dead end
It could still go on. But impractical
 
What precisely went on for pages?
 
My line of thought
For the derivation
Note that I wasn't trying to "prove" Schwarz inequality given it, i was trying to arrive to it by manipulating some other inequality
 
6:32 PM
5 messages moved to ­Trash
 
Hi. I'm looking for feedback on the following question, please.
0
Q: Show, using a specific approach, that $\dim \Bbb P^n=\dim\Bbb A^n=n$.

ShaunThis is Exercise 1.8.4(1) of Springer's, "Linear Algebraic Groups (Second Edition)". It is not a duplicate of The dimension of $\mathbb P^n$ is $n$ because I'm after a particular perspective; namely, the approach Springer takes (using transcendence degree and not Krull dimension). The Question: ...

 
6:51 PM
ChatGPT has problems with simple arithmetic: i.stack.imgur.com/0WsuA.png The meta SO post banning it has almost 120k views in 3 days. meta.stackoverflow.com/q/421831/4014959
@Tink My favourite quote from that Twitter thread is "Because if ChatGPT is, as it seems to be, a consummate bullshitter, it's also—definitionally—a bullshitter who doesn't know when its bullshitting. And we all know that that's the most dangerous kind." — PM 2Ring Dec 5 at 17:27
 
PM i can see the risks in other fields. in early math it might be a useful pedagogical tool. 'spot the error in the proof' type exercises are sometimes too easy, because humans will signal gaps in their understanding with obfuscating language or qualifiers. chatGPT has no qualms. see e.g. chat.stackexchange.com/transcript/message/62510796#62510796
 
Oh, sure. It certainly can be useful for stuff like that.
GPT-3 is very impressive, and it could do lots of useful tasks, like fixing grammatical errors and rephrasing overly-convoluted text.
But it has to be used carefully, not mindlessly, since its grasp on truth and logical consistency is very tenuous.
Here's an amusing dialogue I saw a few months ago: Simulated Elon Musk Lives in a Simulation
The problem witj ChatGPT on SO (& a few other sites) is people were posting large numbers of answers in small timespans, without bothering to verify that the answers were correct, or even self-consistent. And because it writes with good grammar & sentence structure it bypasses some of our usual crap detection methods.
 
7:09 PM
"proof" that there are no odd perfect numbers. sadly this one isn't as "good" as the proof that all finite groups are abelian.
pretty good LaTeX though.
 
I doubt we'd see people posting raw ChatGPT answers on Math.SE, but I wouldn't be surprised to see it on Physics.SE. In fact, I suspect I saw it in action there a couple of weeks ago.
 
I must show that $\lim_{x \to 1^{-1}} \sum_{n=1}^{+\infty} \frac{x^n}{\sqrt{n}}=+\infty$. I tried this: since $\sum_{n=1}^{+\infty}\frac{|x|^n}{\sqrt{n}} \le \sum_{n=1}^{+\infty}|x|^n$ and the latter series is uniformly convergent in any compact $[0,A] \subset [0,1)$, the series $\sum_{n=1}^{+\infty} \frac{x^n}{\sqrt{n}}$ converges uniformly in any compact set $[0,A]$ and so I can exchange the limit and the series when $x \to 1^-$.
Hence $\lim_{x \to 1^-} \sum_{n=1}^{+\infty} \frac{x^n}{\sqrt{n}}=\sum_{n=1}^{+\infty} \lim_{x \to 1^-} \frac{x^n}{\sqrt{n}}=\sum_{n=1}^{+\infty} \frac{1}{\sqrt{n}}=+\infty$. Does this work?
 
PM: there have been some indications of chatbot posts on MSE already. nothing definitive, as far as i know. and nothing resembling the SO scale.
zawarudo: you can't let x go to 1 without leaving every interval of the form [0,A] with A < 1. this might affect your uniform convergence argument, unless by referring to 'exchange the limit and the series' you are referring to something subtler than what i'm thinking of.
 
@leslietownes It's still pretty impressive, and takes a while before it goes off the rails, unlike the one about Z_3 being non-Abelian.
 
and it's typeset so beautifully that it might as well be right :D
 
7:21 PM
I must be doing something wrong, I'm only getting "internal server error"
 
Well, it's good with grammatical structure, so it's not too surprising that it can do decent Latex. Similarly, it usually writes code that doesn't throw syntax errors. And sometimes the code even does the job it's supposed to do. Or it does something quite different. ;)
 
@PM2Ring It gave me beautiful (but VERY VERY WRONG) LaTeX when I asked it to perform polynomial long division.
 
it can mess up integer arithmetic, but typesets it very beautifully. i got 6*3 = 24 or 6*4 = 18 a while ago in something, i forget what.
 
Lukas, the server is possibly overloaded. I suspect that it's getting a lot of traffic.
 
7:25 PM
yeah possibly
 
Howdy, @Xander. I think you scared off that guy with the parabola/envelope. The second post seems also to have vanished — at least, I couldn't find it easily.
 
I only asked it to explain class field theory and it got server errors :(
 
@TedShifrin Well, sest la vee.
 
You meanie.
 
But I didn't do anything. :(
 
7:34 PM
You didn't steal any top secret documents?
 
My question . . .
-1
Q: Show, using a specific approach, that $\dim \Bbb P^n=\dim\Bbb A^n=n$.

ShaunThis is Exercise 1.8.4(1) of Springer's, "Linear Algebraic Groups (Second Edition)". It is not a duplicate of The dimension of $\mathbb P^n$ is $n$ because I'm after a particular perspective; namely, the approach Springer takes (using transcendence degree and not Krull dimension). The Question: ...

 
@TedShifrin Well, sure. But everyone does THAT!
 
. . . is attracting more downvotes.
One comment gave a reason: an answer would take too long.
How am I supposed to know that?
 
@Shaun about that question: does my answer suffice or is there still anything unclear?
 
People can downvote for all kinds of reasons. Maybe they just don't like Krull. Who knows? If they don't bother to explain themselves, move on.
 
7:38 PM
@LukasHeger Yes, it makes sense. I upvoted it. I am making sure I understand it fully before accepting it.
 
BTW, $K(X)$ is the field of rational functions on $X$, as usual, I assume.
 
@XanderHenderson maybe they don't like the fact that Springer didn't publish with Springer. Totally a missed opportunity
2
 
@LukasHeger I mean, that's why I downvoted. :P
 
Just make sure everyone downvotes @Leslie. He has a crummy excuse that he caught covid from his daughter's daycare.
 
7:41 PM
@leslietownes thanks for the answer, so the problem is that even if $0<A<1$ is arbitrary it is fixed before taking the limit and so when $x \to 1^-$ it will be $A<x<1$ eventually and so I lose the uniform convergence?
 
Yes.
 
@Shaun I might have said a few things about transcendence degree that seem to be useful, or were useful in similar problems. Likewise for P^n (i.e. stating whether you know and understand the standard affine cover would be helpful). It's tough (for me) to answer a question in which the asker does not state what they know already.
 
In principle, I totally agree, @Karl. This is why I insist that OPs show me some effort and indicate what's in their toolbox. This is an issue in most areas of mathematics, but very much so in differential and algebraic geometry.
@ZaWarudo Have you tried copying the argument in the proof of Abel's Theorem (normally done with a convergence hypothesis)? The key notion is summation by parts.
 
@KarlKroningfeld Thank you for the feedback. I tried to supply all I knew but, down to ignorance (but not lack of effort), I just couldn't include enough, evidently.
 
@TedShifrin thank you Ted, you're like a fortune teller because I remembered some result from series of function that helped in a situation like this but I couldn't remember the result, so thanks for reading my mind) I will try. So your "yes" was referring to my answer to leslie?
 
7:49 PM
@ZaWarudo if you want to go overkill, note that you can apply Beppo Levi wrt the counting measure, because if we define $f_n(x)=x^n/\sqrt{n}$, then $f_n$ is monotonically decreasing on $(0,1]$
 
@ZaWarudo Yes :)
That exceeds my knowledge, @Lukas.
 
@LukasHeger thank you Lukas but unfortunately I haven't studied that yet :(
 
@TedShifrin I'm sure you know the result, maybe it's not called Beppo Levi in English
if you have monotone convergence of nonnegative functions, you can exchange limit and integral
ofc assuming everything is measurable
and $\infty$ is allowed here
 
@LukasHeger I'll Beppo your Levi!
 
Is that not just the monotone convergence theorem?
I think Beppo Levi is more general.
But, yeah, I suspect ZaWarudo is in a beginning analysis course and doesn't know integration theory.
 
7:52 PM
ah I see so it's called monotone convergence theorem in English
in German we usually say Beppo Levi theorem
 
But from my quick googling, Beppo Levi is more general, in that it does not assume monotonicity.
 
oh okay
 
No, that's wrong. It just doesn't assume nonnegative functions. To be honest, I now forget the monotone convergence that I know.
 
it was just a tangential remark, I'm aware that this is probably not the right powerlevel of solution
 
Why is that matrix of a bilinear form is defined differently than the matrix of a form (sesqui- linear) is defined?
 
7:56 PM
Huh? @Koro
 
@Koro is it?
the representing matrix in both cases is just evaluation at pairs of basis vectors
 
The sesqui only affects the second input vector.
 
it's just that $v^TAw$ defines a bilinear form, whereas $v^TA\overline{w}$ defines a sesquilinear form. Is that your question?
 
You should have different $v$, $w$, or else you're talking quadratic form.
Ah, you read my mind.
 
yeah I was thinking about quadratic forms for a second
 
7:59 PM
How does this solve the following conundrum: Let f be a (sequi) form on finite dimensional vector space V over F. Matrix of f is $(a_{ij}$ w.r.t. some ordered basis $B=\{b_1,...,b_n\}$, where $a_{ij}= f(b_j,b_i)$.
But if f were a bilinear form (say for example F is R), then $a_{ij}:=f(\color{red}{b_i, b_j})$
 
doesn't seem very conundrumous to me
 
What conundrum? Who cares what order they're in?
Some author is doing it differently for some reason.
And who knows whether the conjugate goes on the first variable or on the second.
 
it's seems like a weird choice to do it one way for quadratic forms and the other for sesquilinear forms, I agree on that
 
conjugate goes in the second slot.
 
koro there is no consensus on which side of the sesquilinear form is going to be conjugate linear. they could want the "b_i" part to be the linear part and have the conjugate linearity in the first entry.
 
8:02 PM
Leslie, conjugation part is ok. It's just the matrix of form that's confusing me.
 
well, the bilinear form could be non-symmetric... so it could matter, you could get an additional transposition
 
yeah
 
it's not a mathematical issue, it's just conventions
 
i guess i'd want to see a situation where it comes up in a way that feels like it makes a substantive difference. if this is just a definition looking weird, it just looks weird.
sesquilinearity is just like bilinearity, except when it's not.
 
I'm referring to chapter 9 (where $a_{ij}= f(b_j, b_i)$) and chapter 10 (here arguments of f are reversed to get ij-th entry of its matrix) of Hoffman and Kunze's.
 
8:04 PM
Students always get confused by the order of indices in the equation $T(v_i) = \sum a_{ji}v_j$.
Or $T(v_j) = \sum a_{ij}v_i$.
 
indices are the hardest part in linear algebra
 
Either way, $\sum a_{ij}x_j$ and $\sum a_{ij}v_i$ are different beasts.
@Lukas It's even more fun with multilinear/exterior. I stopped noticing about 40 years ago.
 
The problem is that: Suppose that F= R. Given: f is a form on finite dimensional vector space V, find matrix of f w.r.t. basis B.
 
@Koro WTF cares?
 
@LukasHeger No, the stick I hit students with when the incorrectly perform row operations is the hardest part of linear algebra.
 
8:06 PM
well, the difference will never be substantive, it will just be annoying to remember that you recover a bilinear form via $f(v,w)=v^T A w$ and a sesquilinear form via $f(v,w)=\overline{w}^TAv$, that is all
 
Oh, come on, @Xander. Subtracting a negative wrong isn't the end of the world.
 
there is nothing substantial going on here
 
Note that in this case f is bilinear as well. So get two (transpose of each other) matrices as "matrix of f".
 
Yes, and who cares? This is not a JEE exam anymore.
 
@TedShifrin No, but its a whackin'!
 
8:07 PM
I'm rolling $e^{\pi^e}$ eyes for this one.
 
@TedShifrin Don't be irrational!
 
@Koro we all agree that the conventions are weird, but that's about it
 
(Or ARE YOU?!)
 
@XanderHenderson Are you even sure?
 
8:10 PM
@XanderHenderson It should be a paddlin' for row operations.
 
@Koro I think a more useful convention would be $a_{ij}=42f(b_i,b_j)$, now you have three matrices as "matrices of f"!
 
Unless you're in characteristic 2, 3, or 7. In which case you use 25.
 
obviously
 
I mean in our linear algebra class, an orthonormal basis was not actually a basis! because an orthogonal basis was defined to be ordered and of course an ordered basis is not a basis, it just gives rise to a basis because an $n$-tuple is not necessarily an $n$-element set. Does anyone care? no
 
8:15 PM
Well, ordinarily you are more pedantic than I am, @Lukas.
 
@LukasHeger I care.
Deeply.
 
Is there a good example of a pointwise convergent sequence of holomorphic functions (on the open unit disk, say) whose limit is not holomorphic?
@TedShifrin
 
Oh, I thought about this 15 years ago when I taught grad complex. I had to ask an analysis colleague.
The answer is "yes," but I don't remember it.
What about the real differentiable case?
 
in the real case, every continuous $f:[a,b] \to \Bbb R$ is a limit of polynomials
 
8:25 PM
@TedShifrin In the real case, can't you just consider a sequence of smooth bump functions $\eta_n$ supported on $B(0,1/n)$?
 
In the real case you can have a peak that gets narrower
 
Or, you know, that.
 
@Lukas That's even uniform.
 
yes
 
But smooth bump functions aren't analytic, so...
 
8:27 PM
stone weirstrass :)
 
@Koro Yum...
 
I started writing down an example in the margin of my textbook, but ...
 
I have a feeling the counterexample was in an email that went away when I retired/moved.
 
This has something to do with Osgood, doesn't it?
 
@AkivaWeinberger Last week, you were distributing points uniformly on a sphere. One way to do that is with a Fibonacci spiral. Here's a demo. The epsilon=0.36 (allegedly) makes the mean closest neighbour distance the most uniform.
 
8:32 PM
Nice
I ended up doing that:
I guess I didn't post it here
What is epsilon?
 
That looks pretty good.
My code abbreviates it as esp.
 
let me aks you something about esp
 
Right... some Googling turns up some notes from Krantz: math.wustl.edu/~sk/limits.pdf .
 
Ah, cool!
 
Example 1 seems to be the desired example.
 
8:36 PM
I got the algorithm (with epsilon) from here: extremelearning.com.au/…
 
And, yes, Osgood is the relevant theorem to google.
Clearly, I haven't forgotten everything learned in graduate school. :/
 
I think the question I was wrestling with 15 years ago was something about limits of coefficients of the Taylor expansions of the sequence.
Yes, Xander, you were right, and I don't think I knew Osgood's result.
So, with uniform convergence, the Taylor coefficients converge. But how crazy can they be in the case of pointwise convergence?
 
@leslietownes Oops.
 
he's the one that makes you os all right
 
I don't understand their sets @XanderHenderson
Surely $Re(z)=0$ is included in $|Re(z)|<1-1/[j+2]$
 
8:41 PM
@AkivaWeinberger Whose sets?
 
In the paper you shared
 
Ah.
Hang on.
 
Ohh wait I see
 
(Re z or Im z) AND Im is small AND Re is small
 
8:42 PM
That isn't the point.
Yes!
That.
The picture on the next page is very helpful. :D
 
I don't understand Runge
or why it lets us define those holomorphic functions
Isn't that about approximating an already holomorphic function with rational functions?
Oh, the holomorphic functions in the hypothesis of Runge are piecewise constant here.
I see.
 
The domain matters, is my guess
 
Yes, yes.
 
Related:
Never mind.
So the differences between successive functions in that sequence, then, are bounded functions.
So we have a way to add bounded holomorphic functions in a pointwise convergent way and yet be ill-behaved.
$f_{j+1}-f_j$, I mean.
Can we assume these differences are all bounded by the same amount?
'Cause we can push the poles to pretty far away
and I just care about $U$
 
Hi guys!
 
9:27 PM
Hello, anyone can help me with a question regarding random numbers that have exponential distribution?
I have been trying for the last 3 4 days to find a solution to a problem I am facing, but I haven't gotten anywhere
 
Just ask. Don't ask to ask. :D
 
I am considering a muon decay, basically an exponential pdf
I need to produce random numbers with exp. distri. within a desired interval
I can't think of how I can do that
Considering the particle decay equation $N(t)=N_0e^{-\frac t \tau}$
Integrating this within the interval I, find N_0
so i normalize the pdf
But then i am asked to produce random numbers with exp distribution for a given tau
and these nr. will play the role of a data
I tried many ways but nothing works
 
31
Q: From uniform distribution to exponential distribution and vice-versa

luchonachoThis is probably a trivial question, but my search has been fruitless so far, including this wikipedia article, and the "Compendium of Distributions" document. If $X$ has a uniform distribution, does it mean that $e^X$ follows an exponential distribution? Similarly, if $Y$ follows an exponenti...

 
I know how to do that
in python
either with the random.exponential () or with inverse transformation method, using the inverse of the CDF of the pdf of the distribution
but both give me nr in the range 0 and 1
I will briefly write what it is written
The lifetimes of particles are measured, which are stopped in an absorber after passing through a detector. The decays are again registered by the same detector via the decay products. The theoretical lifetime of the particles is 𝜏= 2µs, which roughly corresponds to the lifetime of the muon. Due to the overlapping of the detector signals, lifetimes smaller than 𝑡min= 1µs cannot be reliably measured.
The measurement electronics are only active up to the point in time 𝑡max= 10 µs. The number of decays registered in this way is only very small with 𝑁=50 registered events, so that in an unbinned maximum likelihood fit (i.e. all data points are considered in the likelihood function, not just the entries in bins of a histogram) an exponential function should be adjusted to the lifetimes measured in the interval [𝑡min,𝑡max].
Write a function that generates 50 exponentially distributed random numbers in the sensitive detector interval [𝑡min,𝑡max].
So initially I write the pdf of the decay :
`def pdf_exp_d(t,tau,tmin,tnax):`
`C = 1/(tau*(np.exp(-t_min/tau)-np.exp(-t_max/tau)))`
` return C*np.exp(-t/tau)`
I don't even know in which category my question falles, that way I can ask there
 
9:52 PM
Good to know, @Xander, that you are alive and well, and merely chose not to respond to my ping. Be well, and I mean that sincerely.
 
@amWhy Which ping?
 
I think Xander is pung out.
 
@Xander The one at 2:50 CST (my time in central standard time) in the mods' chat. It's okay. And @Ted I'm more than pung out.
@XanderHenderson It's fine. feel free to delete my comments, but please know that many of keep noting such protection of some users, often at the expense of others. We're not all "stupid".
@Xander See also my comment in the cafe. You, among others, have wanted me to depart/give up. You'll have your wish shortly.
 
10:38 PM
sigh
 

« first day (4509 days earlier)      last day (522 days later) »